LSAT and Law School Admissions Forum

Get expert LSAT preparation and law school admissions advice from PowerScore Test Preparation.

User avatar
 Dave Killoran
PowerScore Staff
  • PowerScore Staff
  • Posts: 5853
  • Joined: Mar 25, 2011
|
#82642
Complete Question Explanation
(The complete setup for this game can be found here: lsat/viewtopic.php?t=1025)

The correct answer choice is (B).

The condition in the question stem stipulates that two ranch houses face each other. That can only happen with houses 3 and 4. And, of course, if house 4 is a ranch then from the third rule house 2 must be a Tudor:

G3-Q14-d1.png

Houses 1 and 7 are the only uncertainties, and you should look for them in the answers of a Could Be True question. Answer choice (E) addresses house 1, but house 1 can never be a ranch house, so it can be eliminated. Answer choice (B) indicates that house 7 is a split-level house, which is possible, and so answer choice (B) is correct.
You do not have the required permissions to view the files attached to this post.
 cfu1
  • Posts: 15
  • Joined: Mar 28, 2017
|
#33766
Question #14 - I'm not really sure how to figure out the right answer with the given constraints and information from the question. From glancing at the question, it seems like choices A, B, C, and D are all plausible answers?
 Adam Tyson
PowerScore Staff
  • PowerScore Staff
  • Posts: 5153
  • Joined: Apr 14, 2011
|
#33771
Hey cfu1, welcome aboard! From your question, I am guessing that there may be a small problem with your setup. Would you mind describing what you diagrammed for the original set of rules? From there we can do a better job of diagnosing the problem.

Meanwhile, consider this: the rules tell us that house 6 is an S, and they also tell us that ever R has to have at least one T next to it. Be careful here - "adjacent" does not mean the same as "facing" (I bet that's where things went sideways on you!) House 8 is adjacent to house 6, and not adjacent to any other house (house 7 faces house 8, but is not adjacent to it). Knowing that much, without even getting into this specific question, we know for sure that house 8 cannot be an R, because every R must have a T next to it on one side or the other or both. Answer A not only cannot be true in this case, it can never be true!

Share with us your original diagram, and then tell us whether the info I just provided changes anything in your analysis. From there, we'll see where we can go together!

Talk to you soon.
 cfu1
  • Posts: 15
  • Joined: Mar 28, 2017
|
#33774
Ahh, here's my starting diagram:
  • ..... ..... S
    2 ..... 4 ..... 6 ..... 8

    1 ..... 3 ..... 5 ..... 7
    ..... R
Going back and rereading the question, in order to satisfy the condition of the question, I realized that R must be numbers 3 and 4, because as you mentioned before, if R is 8, the condition "every R has at least one T adjacent to it" cannot be satisfied.

So the only possible set-up with this question is:

  • T ..... R ..... S ..... T
    2 ..... 4 ..... 6 ..... 8

    1 ..... 3 ..... 5 ..... 7
    T ..... R ..... T ..... S
Which means (B) is the correct answer.

Haha I feel kinda dumb. I think I was missing bits and pieces of each condition, which was why I couldn't figure out the correct answer.

Thanks for your help and guidance!!
 Adam Tyson
PowerScore Staff
  • PowerScore Staff
  • Posts: 5153
  • Joined: Apr 14, 2011
|
#33780
You're welcome! I do think your local setup has some unwarranted info in it, but you still got the right answer as a result. It looks to me like house 1 can be an S or a T, and house 7 can be an R or an S. Check those against the rules and see if they work for you! You want to do as much as you need to on these local questions, but don't box yourself in with inferences that are not required. With your diagram, if the correct answer was "house 7 is a ranch" it would have looked bad to you, and then where would you be?

Nice job getting past that problem! Keep up the good work!
 cfu1
  • Posts: 15
  • Joined: Mar 28, 2017
|
#33787
I will keep this in mind, thanks so much for your help! :)
User avatar
 canecaldo
  • Posts: 2
  • Joined: Jul 02, 2023
|
#102638
How do we know that 8 is T? That is the only thing I cannot seem to figure out.
 Adam Tyson
PowerScore Staff
  • PowerScore Staff
  • Posts: 5153
  • Joined: Apr 14, 2011
|
#102681
You can find an explanation for that inference in the original setup, canecaldo, which can be found here:

viewtopic.php?f=377&t=1025

The short answer is that since 6 is an S, 8 cannot also be an S, and since every R has a T next to it, 8 also cannot be an R. That leaves only T for position 8.

Get the most out of your LSAT Prep Plus subscription.

Analyze and track your performance with our Testing and Analytics Package.